LSAT and Law School Admissions Forum

Get expert LSAT preparation and law school admissions advice from PowerScore Test Preparation.

 Administrator
PowerScore Staff
  • PowerScore Staff
  • Posts: 8917
  • Joined: Feb 02, 2011
|
#47402
Complete Question Explanation
(The complete setup for this game can be found here: lsat/viewtopic.php?t=17028)

The correct answer choice is (A)

This is an interesting question because our final diagram contained no Not Laws on either the third or fourth positions. The first step, then, is to refer to prior work in order to eliminate answers. Any combination of variables that can successfully be placed third and fourth can be used to eliminate one of the incorrect answers (this is a Cannot Be True question, so each incorrect answer Could Be True; thus, if a combination is shown to be possible, that answer choice can be eliminated).

Question #19 shows that U and R can testify third and fourth, but that combination does not appear as one of the answers.

Question #20 shows that S and U (option #1), U and R (option #2), and U and W (option #2) can each testify third and fourth, which eliminates answer choices (C) and (E).

Of the three remaining answer choices—(A), (B), and (D)—answer choice (A) is the most promising. Why? Let’s examine the surface merits of each, by discussing both of the variables in each answer:

Answer choice (A): The first variable (M) is involved in the third rule, and forces one of two variables (T or W) to be second. This is extremely restrictive.

The second variable (T) is involved in the first and third rules, and from the first rule is one of the options to follow another variable in a block. When T is behind M, T cannot be behind S, and so the first rule then dictates that U must testify after S. From the third rule only T or W can testify immediately before M, so when T is behind M, W must testify immediately before M, or second. Thus, the placement of these two variables leads to great restrictions, possibly great enough to lead to an impossible situation.

Answer choice (B): The first variable (R) is involved in the second rule, and forces U to be first or second. This is restrictive, but not as restrictive as the first variable in answer choice (A).

The second variable (S), is involved in the first rule, but that variable is placed in such a position that it can still meet the condition of the first rule without restriction. This is also less restrictive than the second variable in answer choice (A).

Answer choice (D): The first variable (T) is from a block rule, but it is placed in such a position that it can still meet the condition of the first rule without restriction.

The second variable (R) is involved in the sequence, and while this forces U to testify first or second, it is not as restrictive as the second variable placement in (A).

So, if we were using hypotheticals to solve this question, answer choice (A) would be the best starting point. Some might say that you would naturally start at (A) anyway, but that is beside the point. In this question they happen to be the same, but in another question it might be (D) that is the best starting point. Having that information would then save a considerable amount of time in making hypotheticals. Let’s now look at answer choice (A) and see what occurs when M testifies third and T testifies fourth:
D10_game #4_#22_diagram 1.png
As mentioned previously, this scenario forces W to be second, and also creates an SU block:
D10_game #4_#22_diagram 2.png
At this point, the only placement option for the SU block is fifth and sixth, which is impossible due to the requirements of the second rule. Thus, M and T cannot testify third and fourth because that leads to an impossible outcome, and therefore answer choice (A) cannot be true and is correct.
You do not have the required permissions to view the files attached to this post.

Get the most out of your LSAT Prep Plus subscription.

Analyze and track your performance with our Testing and Analytics Package.